4
$\开始组$

考虑一下这个MSE公司问题设置如下:我们有一个集合X美元$具有基数美元=n$和一个家庭$\mathcal{F}$属于$4-$的子集X美元$这样,对于两个不同的$A,B\in\mathcal{F}$,$|A\cap B|\leq 1美元$这样一个家庭能有多大?此处给出的答案暗示了$1+n+\binom{n}{2}$使用Frankl和Wilson的更一般的定理。我们如何直接证明二次(也许是改进的)上界?

对于下限,我只能做一个简单的2016年2月2日$绑定:LetX美元=[n]$1美元(x)=ax+b$,定义集合$S_{l}$成为$\{l(0),l(1),l[2],l(3)\}$然后是家人$\mathcal{F}=\{S_{ax+b}|a,b\in[n/4]\}$满足相交条件,因为两条直线只能在一个点相交,并且$\mathcal{F}$具有大小2016年2月2日$.

$\端组$
  • $\开始组$ 第二部分中定义的$\mathcal{F}$实际上是一个族(两条不同的线对应不同的点集),因为斜率都是正的。 $\端组$ 评论 5月23日17:15
  • $\开始组$ 小$n$的最佳值如下:oeis.org/A004037 $\端组$ 评论 5月23日19:12
  • $\开始组$ 这个答案是有帮助的。 $\端组$ 评论 5月24日1:15

2个答案2

重置为默认值
1
$\开始组$

你的上限是最好的,渐进的。我会告诉你如何在什么时候达到你的极限$n=4^k$,对于一些$k\in\mathbb N$.之后,我给出了一些与其他引用有关的引用n美元$.

首先,考虑一个更难的问题。条件$|A\cap B|\le 1美元$意味着每对元素X美元$最多出现在一个子集中。对于其中n美元$我们能找到一个家庭吗$4$-元素子集X美元$这样,每对元素X美元$一起出现在确切地一个子集?这样的家庭被称为平衡不完全区组设计,并表示为$\mathrm{BIBD}(n,4,1)$.这里的块数必须是$\binom美元{n} 2个/\二进制数{4}2$,因此此设计可以最佳地解决您的问题。

引理:对于所有人$n\in\mathbb n$,如果是$\mathrm{BIBD}(n,4,1)$存在,然后$\mathrm{BIBD}(4n,4,1)$存在。

证明: $\newcommand\s{\spadesuit}\新命令{\h}{\heartsuit}\新命令{\c}{\clubsuit}\新命令{\d}{\diamondsuit}$$\mathcal B公司$是大小的集合-$4$的子集【n】美元$。我将演示如何构造集合$\mathcal C美元$大小为-$4$的子集$\{\s,\h,\c,\d\}\次[n]$。为此,我将使用一对正交拉丁方L美元$百万美元$侧面宽度的n美元$.这样一个百万美元$ 存在为所有人n美元$除了$2$$6$,但一个$\mathrm{BIBD}(n,4,1)$$n\in\{2,6\}$无论如何。

对于每个$B\in\mathcal B$,$\mathcal C美元$包含集合$B\times\{\s\}、B\times \{h\}和B\time\{c\}$、和$B\次\{\d\}$此外,对于每个$i,j\英寸[n]$,$\mathcal C美元$包含集合$$\{(\ s,i),(\ h,j),(\c,L_{i,j})$$您可以使用正交拉丁方的定义来表示$\{\s,\h,\c,\d\}\次[n]$恰好出现在所描述的集合之一中。$\tag*{$\square$}$

现在,通过反复应用这个引理,可以得到一个$\mathrm{BIBD}(4^k,4,1)$对所有人都存在$k\in\mathbb N$因此$\binom n2/\binom 42美元$经常可以无限地实现。

事实上,该界限在加性因子范围内是正确的。如CRC组合设计手册(定理$40.12$),对于所有人$n\ge 4美元$,$$\存在\mathcal F:\quad |\mathcall F|\ge\left\lfloor\frac{n}{4}\left\floor\frac{n-1}三\right\rfloor\right\rploor-3。$$然而,证明这一点的结构相当复杂,所以我不会在这里展示它们。对于以下情况下的结构$n\等于1$$4\pmod{12}$,参见下文引用的Hannani的论文。在这些情况下,Hanani构建了一个$\mathrm{BIBD}(n,4,1)$最后,您可能对下面的Rodl论文感兴趣,该论文展示了如何实现$(1+o(1))\cdot\binom{n} 2个/\二进制数{4}2$使用概率方法。

Charles J.Colbourn和Jeffrey H.Dinitz。2006.组合设计手册,第二版(离散数学及其应用)。查普曼和霍尔/CRC。

H.Hanani,平衡不完全块设计和相关设计,离散数学。11(1975) 255–369.https://doi.org/10.1016/0012-365X(75)90040-0

Vojtěch Rödl,关于包装和覆盖问题,欧洲组合数学杂志,第6卷,第1期,1985年,69-78,ISSN 0195-6698,https://doi.org/10.1016/S0195-6698(85)80023-8.

$\端组$
0
$\开始组$

似乎有一个简单的上限2012年2月2日$:如果$\{S_1,\ldot,S_m\}$那么是家人吗$m\binom{4}{2}\leq\binom}{n}{2{$因为S_j美元$的必须分离。

$\端组$

你必须登录来回答这个问题。

不是你想要的答案吗?浏览标记的其他问题.